Đến nội dung

Hình ảnh

Topic: [LTDH] Mỗi ngày hai bất đẳng thức.


  • Chủ đề bị khóa Chủ đề bị khóa
Chủ đề này có 215 trả lời

#81
MrS

MrS

    Binh nhất

  • Thành viên
  • 29 Bài viết

Giúp em nốt bài này nhé! Cho a,b,c>0,abc=1,tìm max của \sum \frac{a}{a+b^2+c^2}

Ta sẽ chứng minh $\sum \frac{a}{a+b^2+c^2}\leq 1\Leftrightarrow P=\sum \frac{b^2+c^2}{a+b^2+c^2}\geq 2$

Chú ý theo BĐT Cauchy - Schwarz thì: $b^2+c^2\geq \frac{(b+c)^2}{2}$ nên $P=\sum \frac{b^2+c^2}{a+b^2+c^2}\geq \sum \frac{\frac{(b+c)^2}{2}}{a+\frac{(b+c)^2}{2}}=\sum \frac{(b+c)^2}{2a+(b+c)^2}\geq \frac{4(a+b+c)^2)}{2\sum a+\sum (b+c)^2}=A$

Ta sẽ chứng minh $A\geq 2$ hay $2(a+b+c)^2\geq 2\sum a+\sum (b+c)^2\Leftrightarrow ab+bc+ca\geq a+b+c\Leftrightarrow f(a,b,c)=ab+bc+ca-a-b-c\geq 0$

Xét $d=f(a,b,c)-f(a,\sqrt{bc},\sqrt{bc})=(\sqrt{b}-\sqrt{c})^2(a-1)$

Không mất tổng quát giả sử $a=max(a,b,c)$ khi đó $a\geq 1$ suy ra $d\geq 0$

Vậy ta chỉ cần chứng minh $f(a,t,t)=t^2+2at-a-2t\geq 0$ với $t=\sqrt{bc}$  từ điều kiện $at^2=1\Rightarrow a=\frac{1}{t^2}$ thế vào rút gọn ta được $f(a,t,t)=\frac{(t-1)^2(t+1)}{t^2}\geq 0$ với mọi $t> 0$

Bắt đẳng thức được chứng minh

Vậy $MaxP=1$ tại $a=b=c=1$.


Bài viết đã được chỉnh sửa nội dung bởi MrS: 02-09-2016 - 23:16


#82
tritanngo99

tritanngo99

    Đại úy

  • Điều hành viên THPT
  • 1644 Bài viết

Mặc dù bộ giáo dục và đào tạo có những thay đổi về cách thi môn toán , nhưng với lòng đam mê và nhiệt huyết với toán học, đặc biệt là bất đẳng thức, mình xin tiếp tục topic:

Bài 41: Cho $x,y,z$ là các số thực dương thỏa mãn: $5(x^2+y^2+z^2)=6(xy+yz+zx)$.Tìm giá trị nhỏ nhất của biểu thức: $P=(x+y+z)(\frac{1}{x}+\frac{1}{y}+\frac{1}{z})$.

Bài 42: Cho các số thực không âm $x,y$ thỏa mãn: $x^2+y^2+xy+2=3(x+y)$. Tìm giá trị lớn nhất của biểu thức: $P=\frac{3x+2y+1}{x+y+6}$



#83
tritanngo99

tritanngo99

    Đại úy

  • Điều hành viên THPT
  • 1644 Bài viết

Dưới đây là lời giải bài 41 và 42:

Lời giải bài 41: Giả thiết: $5(x+y+z)^2=16(xy+yz+zx)$.

Đặt: $(x;y;z)\to (ka;kb;kc)$ với $k=\frac{x+y+z}{4}$

$\implies a+b+c=4;ab+bc+ca=5$

(Thực chất đây chỉ là kĩ thuật chuẩn hóa).

Ta sẽ tìm GTNN của $P=(a+b+c)(\frac{1}{a}+\frac{1}{b}+\frac{1}{c})=\frac{20}{abc}$.

Vậy nên ta tìm GTLN của $abc$.

Ta có: $abc=c(5-c(a+b))=c^3-4c^2+5c$.

Lại có: $c^2-4c+5=ab\le \frac{(a+b)^2}{4}=\frac{(4-c)^2}{4}$.

Nên $\frac{2}{3}\le c\le 2$.

Khảo sát hàm $f(c)=c^3-4c^2+5c,c\in [\frac{2}{3};2]$.

Ta có kết quả: $Max_{f(c)}=2\iff c=1$.

Vậy $GTNN$ của $P=10$. Dấu $=$ xảy ra tại $x=2y=2z$.

Lời giải bài 42:

 Đặt: $\left\{\begin{matrix} x+y=2a\\x-y=2b  \end{matrix}\right.\implies \left\{\begin{matrix} a>0\\x=a+b\\y=a-b  \end{matrix}\right.$

Khi đó giả thiết trở thành: $(a+b)^2+(a+b)(a-b)+(a-b)^2+2=6a\iff 3a^2+b^2+2=6a$

$\implies P=\frac{5a+b+1}{2a+6}$.

Do $3a^2+b^2+2=6a\implies b^2=-3a^2+6a-2=1-3(a-1)^2\iff |b|=\sqrt{1-3(a-1)^2}$.

Từ đó suy ra: $P=\frac{5a+b+1}{2a+6}\le \frac{5a+|b|+1}{2a+6}=\frac{5a+1+\sqrt{1-3(a-1)^2}}{2a+6}$.

Đặt $a-1=t$ thì $t\in [\frac{-1}{\sqrt{3}};\frac{1}{\sqrt{3}}]$.

Ta viết lại $P=\frac{5(t+1)+1+\sqrt{1-3t^2}}{2(t+1)+6}=\frac{5t+6+\sqrt{1-3t^2}}{2t+8}$.

Đến đây xét hàm $f(t)\forall t\in [\frac{-1}{\sqrt{3}};\frac{1}{\sqrt{3}}]\implies f(t)\le 1$. Dấu $=$ xảy ra tại $t=\frac{1}{2}$.

Lời giải hoàn tất. Dấu $=$ xảy ra khi $x=2;y=1$.

 



#84
tritanngo99

tritanngo99

    Đại úy

  • Điều hành viên THPT
  • 1644 Bài viết

Tiếp theo:

Bài 43: Cho $a,b,c>0$ và thõa mãn: $a+b+c=1$. Chứng minh rằng:

$\sqrt{a+\frac{(b-c)^2}{4}}+\sqrt{b}+\sqrt{c}\le \sqrt{3}$.

Bài 44: Với các số thực dương $x,y,z$ thỏa mãn: $x+y+1=z$. Tìm GTNN của biểu thức:

$P=\frac{x^3}{x+yz}+\frac{y^3}{y+zx}+\frac{z^3}{z+xy}+\frac{14}{(z+1)\sqrt{(x+1)(y+1)}}$



#85
Lequynhdiep

Lequynhdiep

    Binh nhì

  • Thành viên mới
  • 13 Bài viết

Cho a,b,c >0 thỏa mãn : abc =1 . c/m r: a+ b+ c$\geq a\sqrt{b+c} +b\sqrt{c+a} + c\sqrt{a+b}$



#86
ecchi123

ecchi123

    Trung sĩ

  • Điều hành viên OLYMPIC
  • 177 Bài viết

Tiếp theo:

Bài 43: Cho $a,b,c>0$ và thõa mãn: $a+b+c=1$. Chứng minh rằng:

$\sqrt{a+\frac{(b-c)^2}{4}}+\sqrt{b}+\sqrt{c}\le \sqrt{3}$.

Bài 44: Với các số thực dương $x,y,z$ thỏa mãn: $x+y+1=z$. Tìm GTNN của biểu thức:

$P=\frac{x^3}{x+yz}+\frac{y^3}{y+zx}+\frac{z^3}{z+xy}+\frac{14}{(z+1)\sqrt{(x+1)(y+1)}}$

 Bài 43 : ( học sinh giỏi nữ sinh trung quốc năm bao nhiêu em không nhớ ) lời giải đại loại như sau :

$VT^2 \leq 3(a+\frac{(c-b)^2}{4}+\frac{(\sqrt{b}+\sqrt{c})^2}{4}+\frac{(\sqrt{b}+\sqrt{c})^2}{4})$

=$3(a+\frac{c+b}{2}+\frac{(b-c)^2+4\sqrt{bc}}{4})\leq 3(a+\frac{b+c}{2}+\frac{(b+c)(\sqrt{b}-\sqrt{c})^2}{2}+\sqrt{bc})\leq 3(a+\frac{b+c}{2}+\frac{(\sqrt{b}-\sqrt{c})^2}{2}+\sqrt{bc})=3$ do đó cod điề phải cm đẳng thức xảy ra khi 3 biến bằng nhau


Bài viết đã được chỉnh sửa nội dung bởi ecchi123: 17-09-2016 - 22:36

~O)  ~O)  ~O)


#87
hanguyen445

hanguyen445

    Thượng sĩ

  • Thành viên
  • 240 Bài viết

Tiếp theo:

Bài 43: Cho $a,b,c>0$ và thõa mãn: $a+b+c=1$. Chứng minh rằng:

$\sqrt{a+\frac{(b-c)^2}{4}}+\sqrt{b}+\sqrt{c}\le \sqrt{3}$.

Bài 44: Với các số thực dương $x,y,z$ thỏa mãn: $x+y+1=z$. Tìm GTNN của biểu thức:

$P=\frac{x^3}{x+yz}+\frac{y^3}{y+zx}+\frac{z^3}{z+xy}+\frac{14}{(z+1)\sqrt{(x+1)(y+1)}}$

 Bài tập 11:Cho $x,y,z>0:x+y+1=z$. Tìm giá trị nhỏ nhất biểu thức sau

 $P=\dfrac{x^3}{x+yz}+\dfrac{y^3}{y+zx}+\dfrac{z^3}{z+xy}+\dfrac{14}{(z+1)\sqrt{(x+1)(y+1)}}$

$\bullet$\textbf{Nhận xét: Bài toán đối xứng 2 biến $y,z$ nên dự đoán điểm rơi $y=z$}

Sử dụng BĐT phụ $\dfrac{a^3}{x}+\dfrac{b^3}{y}\ge\dfrac{(a+b)^3}{2(x+y)}$ ta :

$\dfrac{x^3}{x+yz}+\dfrac{y^3}{y+zx}\ge\dfrac{(x+y)^3}{2(x+y+z(x+y))}=\dfrac{(x+y)^2}{2(z+1)}$

 Thứ hai $x+y+1=z\Leftrightarrow z+1=(x+1)+(y+1)\ge 2\sqrt{(x+1)(y+1)}\Rightarrow (z+1)^2\ge 4(x+1)(y+1)$

 Thứ ba $x+y+1=z\Rightarrow z+xy=xy+x+y+1=(x+1)(y+1)\le\dfrac{(z+1)^2}{4}$

Suy ra: $P\ge\dfrac{(z-1)^2}{2(z+1)}+\dfrac{4z^3}{(z+1)^2}+\dfrac{28}{(z+1)^2}=\dfrac{9z^3-z^2-z-57}{2(z+1)^2}=f(z)$

Khảo sát hàm $f(z)$ với $z>0$:

+Ta $f'(z)=0\Leftrightarrow z=\dfrac{5}{3}\Rightarrow f(z)\ge\dfrac{53}{8}$

+ Vậy $Min$ P=$\dfrac{53}{8}$ khi $x=y=\dfrac{1}{3};z=\dfrac{5}{3}$


Bài viết đã được chỉnh sửa nội dung bởi hanguyen445: 17-09-2016 - 23:55

Đề thi chọn đội tuyển  HSG:

http://diendantoanho...date-2016-2017/

Topic thảo luận bài toán thầy Hùng:

http://diendantoanho...topicfilter=all

Blog Thầy Trần Quang Hùng

http://analgeomatica.blogspot.com/

Hình học: Nguyễn Văn Linh

https://nguyenvanlin...ss.com/2016/09/

Toán học tuổi trẻ:

http://www.luyenthit...chi-thtt-online

Mathlink:http://artofproblemsolving.com

BẤT ĐẲNG THỨC:

http://diendantoanho...-đẳng-thức-vmf/

http://diendantoanho...i-toán-quốc-tế/

 


#88
tritanngo99

tritanngo99

    Đại úy

  • Điều hành viên THPT
  • 1644 Bài viết

Cho a,b,c >0 thỏa mãn : abc =1 . c/m r: a+ b+ c$\geq a\sqrt{b+c} +b\sqrt{c+a} + c\sqrt{a+b}$

Do Lequynhdiep là thành viên mới, chưa đọc rõ nội quy nên mình xem đây như trường hợp đặc biệt 2 của Topic và xem đây như bài 44*. Hi vọng lần sau bạn chú ý hơn.

Đề đúng của bài toán này là:

Cho $a,b,c>0$ thỏa mãn: $abc=2$. Chứng minh rằng: $a^3+b^3+c^3\ge a\sqrt{b+c}+b\sqrt{c+a}+c\sqrt{a+b}$.

Lời giải bài 44*:

Không mất tính tổng quát: Giả sử: $a\ge b\ge c$.

Áp dụng BDT Trêbưsêp ta có:

$a^3+b^3+c^3\ge \frac{1}{3}(a^2+b^2+c^2)(a+b+c)(1)$.

Theo $AM-GM$ ta có: $a^3+b^3+c^3\ge 3abc=6(2)$.

Nhân $(1),(2)$ vế theo vế ta được:

$(a^3+b^3+c^3)^2\ge 2(a^2+b^2+c^2)(a+b+c)\implies a^3+b^3+c^3\ge \sqrt{(a^2+b^2+c^2)(a+b+c)}$.

Áp dụng BDT BCS ta có:

$\sqrt{2(a^2+b^2+c^2)(a+b+c)}=\sqrt{(a^2+b^2+c^2)(a+b+b+c+c+a)}\ge a\sqrt{b+c}+b\sqrt{c+a}+c\sqrt{a+b}\implies Q.E.D$.

Dấu $=$ xảy ra tại $a=b=c=\sqrt[3]{2}$. 

Ps: Tổng quát của $(1)$.

Bài toán tổng quát: Cho $a,b,c$ là những số thực dương. Chứng minh rằng bất đẳng thức sau luôn đúng:

$a^{m+n}+b^{m+n}+c^{m+n}\ge \frac{1}{3}(a^m+b^m+c^m)(a^n+b^n+c^n)(*)$.

Chứng minh: $(*)\iff \sum (a^m-b^m)(a^n-b^n)\ge 0(TRUE)\implies Q.E.D$


Bài viết đã được chỉnh sửa nội dung bởi tritanngo99: 19-09-2016 - 18:11


#89
hanguyen445

hanguyen445

    Thượng sĩ

  • Thành viên
  • 240 Bài viết

Ủng hộ bài đầu tiên  :D

Giả thiết bài toán có thể viết lại thành: $(x+y)(x+z)=4yz$.

Áp dụng bất đẳng thức Bunyakovsky, ta có:

$(x+y)(x+z)\geq (x+\sqrt{yz})^2\Rightarrow 4yz\geq (x+\sqrt{yz})^2\Rightarrow x\leq \sqrt{yz}$

 

Vậy ta có:

 

$(x+y)^3+(x+z)^3\leq (y+\sqrt{yz})^3+(z+\sqrt{yz})^3$

 

$=(\sqrt{y}+\sqrt{z})^3((\sqrt{y})^3+(\sqrt{z})^3)=(\sqrt{y}+\sqrt{z})^4(y-\sqrt{yz}+z)$

 

$=\frac{1}{4}(y+2\sqrt{yz}+z)^2(4y-4\sqrt{yz}+4z)\leq \frac{1}{4}(\frac{4y-4\sqrt{yz}+4z+2(y+2\sqrt{yz}+z)}{3})^3=2(y+z)^3$

 

Và ta cũng có: $3(x+y)(y+z)(z+x)=12yz(y+z)\leq 3(y+z)^3$. Do ta có bất đẳng thức quen thuộc $(y+z)^3\geq 4yz(y+z)\Leftrightarrow 3(y-z)^2(y+z)\geq 0$

Cộng hai bất đẳng thức trên và ta được $(x+z)^3+(x+y)^3+3(x+y)(y+z)(z+x)\leq 5(y+z)^3$.

Nên có đpcm.

\textit{Lời giải[2]: Dồn biến về $y+z$

Từ giả thiết: $x(x+y+z)=3yz\Rightarrow\begin{cases}x^2+x(y+z)=3yz\Rightarrow x\le\sqrt{yz}\le\dfrac{y+z}{2}\\(x+y)(y+z)=4yz\end{cases}$

\\ ($\bullet$ )\textbf{Hướng 1:} Dồn biến về trung bình công hay trung bình nhân đều nhận thất bại. Tức :

\\ $P=f(x)\le f(\sqrt{xy})\le f(\dfrac{x+y}{2})>5(y+z)^3$ (ngược dấu).

\\ ($\bullet$ )\textbf{Hướng 2:} Cũng dồn biến về $y+z$. Tuy nhiên để đảm bảo độ chặt BĐT ta chưa vội đánh giá, đi biến đổi BĐT để thể thay tối đa giả thiết, nhằm hạn chế tối thiểu việc đánh giá BĐT thức. Khi đó mức độ thành công bài toán cao hơn. Thật vậy:}

\\ +$ Q=(x+y)^3+(x+z)^3=2x^3+y^3+z^3+3xy(x+y)+3xz(x+z)$

$=2x^3+y^3+z^3+3x^2(y+z)+3x(y^2+z^2)=2x[x(x+y+z)]+x^2(y+z)+3x(y^2+z^2)+y^3+z^3$

$=6xyz+x^2(y+z)+3x(y^2+z^2)+y^3+z^3=3x(y+z)^2+y^3+z^3+x^2(y+z)$\\

$=(y+z)[x(x+y+z)]+2x(y+z)^2+y^3+z^3=3yz(y+z)+y^3+z^3+2x(y+z)^2$

\\ $=(y+z)^3+2x(y+z)^2$\\

\\ +$T=3(x+y)(x+z)(y+z)=12yz(y+z)$.

\\ Do đó BĐT được cần chứng minh tương đương với:

\\ \centerline{$\Leftrightarrow (y+z)^3+2x(y+z)^2+12yz(y+z)\le 5(y+z)^3$}\\

\centerline{$\Leftrightarrow (y+z)(y+z-2x)+3(y-z)^2\ge 0$ điều này l.đúng}.

\\ Vậy BĐT được chứng minh.\\


Bài viết đã được chỉnh sửa nội dung bởi hanguyen445: 18-09-2016 - 10:01

Đề thi chọn đội tuyển  HSG:

http://diendantoanho...date-2016-2017/

Topic thảo luận bài toán thầy Hùng:

http://diendantoanho...topicfilter=all

Blog Thầy Trần Quang Hùng

http://analgeomatica.blogspot.com/

Hình học: Nguyễn Văn Linh

https://nguyenvanlin...ss.com/2016/09/

Toán học tuổi trẻ:

http://www.luyenthit...chi-thtt-online

Mathlink:http://artofproblemsolving.com

BẤT ĐẲNG THỨC:

http://diendantoanho...-đẳng-thức-vmf/

http://diendantoanho...i-toán-quốc-tế/

 


#90
hanguyen445

hanguyen445

    Thượng sĩ

  • Thành viên
  • 240 Bài viết

Theo nguyên tắc Đi-rích-lê thì tồn tại 2 số cùng lớn hơn hoặc nhỏ hơn 1.

Giả sử $(a-1)(b-1)\geq 0\Leftrightarrow ab\geq a+b-1\Leftrightarrow abc\geq c(a+b)-c=c(3-c)-c=2c-c^{2}$

Do đó ta có:

$3(a^{2}+b^{2}+c^{2})+4abc\geq \frac{3}{2}(a+b)^{2}+3c^{2}+4(2c-c^{2})=\frac{3}{2}(3-c)^{2}+3c^{2}+4(2c-c^{2})=\frac{(c-1)^{2}+26}{2}\geq 13$

Dấu = xảy ra$\Leftrightarrow a=b=c=1$

 Thuần nhấtBĐT $\Leftrightarrow (a+b+c)(a^+b^2+c^2)+4abc\ge\dfrac{13}{27}(a+b+c)^3\Leftrightarrow 14(a^3+b^3+c^3)+30abc\ge 12\sum a(b^2+c^2)$

$\Leftrightarrow 14(a^3+b^3+c^3+3abc)\ge 12\sum a(b^2+c^2)+12abc$

 Theo BĐT schur: $a^3+b^3+c^3+3abc\ge\sum a(b^2+c^2)$

 Suy ra BĐT đúng nếu ta chứng minh được:

 $2\sum a(b^2+c^2)\ge 12abc\Leftrightarrow\sum a(b-c)^2\ge 0$, luôn đúng. Do đó BĐT được chứng minh.


Đề thi chọn đội tuyển  HSG:

http://diendantoanho...date-2016-2017/

Topic thảo luận bài toán thầy Hùng:

http://diendantoanho...topicfilter=all

Blog Thầy Trần Quang Hùng

http://analgeomatica.blogspot.com/

Hình học: Nguyễn Văn Linh

https://nguyenvanlin...ss.com/2016/09/

Toán học tuổi trẻ:

http://www.luyenthit...chi-thtt-online

Mathlink:http://artofproblemsolving.com

BẤT ĐẲNG THỨC:

http://diendantoanho...-đẳng-thức-vmf/

http://diendantoanho...i-toán-quốc-tế/

 


#91
hanguyen445

hanguyen445

    Thượng sĩ

  • Thành viên
  • 240 Bài viết

mathtype 

 

Hình gửi kèm

  • sa.JPG

Bài viết đã được chỉnh sửa nội dung bởi hanguyen445: 18-09-2016 - 16:08

Đề thi chọn đội tuyển  HSG:

http://diendantoanho...date-2016-2017/

Topic thảo luận bài toán thầy Hùng:

http://diendantoanho...topicfilter=all

Blog Thầy Trần Quang Hùng

http://analgeomatica.blogspot.com/

Hình học: Nguyễn Văn Linh

https://nguyenvanlin...ss.com/2016/09/

Toán học tuổi trẻ:

http://www.luyenthit...chi-thtt-online

Mathlink:http://artofproblemsolving.com

BẤT ĐẲNG THỨC:

http://diendantoanho...-đẳng-thức-vmf/

http://diendantoanho...i-toán-quốc-tế/

 


#92
hanguyen445

hanguyen445

    Thượng sĩ

  • Thành viên
  • 240 Bài viết

del

Bài viết đã được chỉnh sửa nội dung bởi hanguyen445: 18-09-2016 - 19:56

Đề thi chọn đội tuyển  HSG:

http://diendantoanho...date-2016-2017/

Topic thảo luận bài toán thầy Hùng:

http://diendantoanho...topicfilter=all

Blog Thầy Trần Quang Hùng

http://analgeomatica.blogspot.com/

Hình học: Nguyễn Văn Linh

https://nguyenvanlin...ss.com/2016/09/

Toán học tuổi trẻ:

http://www.luyenthit...chi-thtt-online

Mathlink:http://artofproblemsolving.com

BẤT ĐẲNG THỨC:

http://diendantoanho...-đẳng-thức-vmf/

http://diendantoanho...i-toán-quốc-tế/

 


#93
tritanngo99

tritanngo99

    Đại úy

  • Điều hành viên THPT
  • 1644 Bài viết

Các bài 43 và 44 các bạn đã cho lời giải đúng, mình xin tiếp tục:

Bài 45: Cho $a,b,c$ là các số thực dương thỏa mãn: $a+b-c=1$. Tìm giá trị nhỏ nhất:

$P=\sqrt[3]{\frac{9a^2}{(b+c)^2+5bc}}+\sqrt[3]{\frac{9b^2}{(c+a)^2+5ca}}+\frac{a+b+c^2}{9}$.

Bài 46: Cho các số thực dương $a,b,c$ thỏa mãn: $c=min(a,b,c)$ và $a^2+b^2+c^2=3$. Tìm GTLN của:

$P=2ab+3bc+3ca+\frac{6}{a+b+c}$


Bài viết đã được chỉnh sửa nội dung bởi tritanngo99: 18-09-2016 - 19:38


#94
hanguyen445

hanguyen445

    Thượng sĩ

  • Thành viên
  • 240 Bài viết

Lời giải không được đẹp mắt

Hình gửi kèm

  • 23.JPG

Đề thi chọn đội tuyển  HSG:

http://diendantoanho...date-2016-2017/

Topic thảo luận bài toán thầy Hùng:

http://diendantoanho...topicfilter=all

Blog Thầy Trần Quang Hùng

http://analgeomatica.blogspot.com/

Hình học: Nguyễn Văn Linh

https://nguyenvanlin...ss.com/2016/09/

Toán học tuổi trẻ:

http://www.luyenthit...chi-thtt-online

Mathlink:http://artofproblemsolving.com

BẤT ĐẲNG THỨC:

http://diendantoanho...-đẳng-thức-vmf/

http://diendantoanho...i-toán-quốc-tế/

 


#95
hanguyen445

hanguyen445

    Thượng sĩ

  • Thành viên
  • 240 Bài viết

Bài tập 45

Hình gửi kèm

  • wq.JPG

Đề thi chọn đội tuyển  HSG:

http://diendantoanho...date-2016-2017/

Topic thảo luận bài toán thầy Hùng:

http://diendantoanho...topicfilter=all

Blog Thầy Trần Quang Hùng

http://analgeomatica.blogspot.com/

Hình học: Nguyễn Văn Linh

https://nguyenvanlin...ss.com/2016/09/

Toán học tuổi trẻ:

http://www.luyenthit...chi-thtt-online

Mathlink:http://artofproblemsolving.com

BẤT ĐẲNG THỨC:

http://diendantoanho...-đẳng-thức-vmf/

http://diendantoanho...i-toán-quốc-tế/

 


#96
tritanngo99

tritanngo99

    Đại úy

  • Điều hành viên THPT
  • 1644 Bài viết

Xin cảm ơn bạn hanguyen445 rất nhiều về lời giải hay của hai bài 45 và bài 46:

Dưới đây là 2 lời giải khác của bài 45 và bài 46:

Lời giải bài 45:

Dùng bất đẳng thức $AG-GM$:  $bc\le \frac{(b+c)^2}{4}$.

Ta đánh giá được: $P\ge (\frac{2a}{b+c})^{\frac{2}{3}}+(\frac{2b}{c+a})^{\frac{2}{3}}+\frac{c^2+c2+1}{9}$.

Ta có một đánh giá khá nổi tiếng và chặt như sau: 

$(\frac{2a}{b+c})^2-(\frac{3a}{a+b+c})^3=\frac{a^2(a+4b+4c)(2a-b-c)^2}{(b+c)^2(a+b+c)^3}\ge 0$.

(Các bạn cần chú ý và xem đây như một bổ đề quan trọng).

Vậy nên: $(\frac{2a}{b+c})^{\frac{2}{3}}\ge \frac{3a}{a+b+c}$.

Tương tự: $(\frac{2b}{c+a})^{\frac{2}{3}}\ge \frac{3b}{a+b+c}$.

Vậy $P\ge \frac{3c+3}{2c+1}+\frac{c^2+c+1}{9}=\frac{7}{3}+\frac{(2c+7)(c-1)^2}{9(2c+1)}\ge \frac{7}{3}$.

Vậy $Min(P)=\frac{7}{3}$.Dấu $=$ xảy ra khi $a=b=c$.

Lời giải bài 46:

Ta có đánh giá như sau: $2ab+3bc+3ca\le (a+b+c)^2-1(*)$

Ta có: $(a+b+c)^2-1-2ab-3bc-3ca=2-bc-ca=2-c(a+b)\ge 2-c\sqrt{2(a^2+b^2)}=2-c\sqrt{2(3-c^2)}$

$=2-\sqrt{2c^2(3-c^2)}\ge 2-\frac{2c^2+3-c^2}{2}=\frac{1-c^2}{2}\ge 0$.

Áp dụng, ta suy ra: $P\le (a+b+c)^2+\frac{6}{a+b+c}-1$.

Đặt $t=a+b+c,(\sqrt{3}<t\le 3)$.

Khi đó: $P\le f(t)=t^2+\frac{6}{t}-1$.

Xét hàm số: $f(t)=t^2+\frac{6}{t}-1,t\in (\sqrt{3};3]$.

Ta có: $f'(t)=2t-\frac{6}{t^2}>0,\forall t\in (\sqrt{3};3]\implies P\le f(t)\le f(3)=10$.

Lời giải hoàn tất. Dấu đẳng thức xảy ra tại $a=b=c=1$.

Ps: Đánh giá $(*)$ có lẽ là điều chúng ta đáng nghi vấn nhất ở lời giải này. Phần này mình để các bạn suy nghĩ thử?



#97
tritanngo99

tritanngo99

    Đại úy

  • Điều hành viên THPT
  • 1644 Bài viết

Tiếp theo:

Bài 47: Cho $a,b,c$ là các số thực dương thỏa mãn: $(3a+2b+c)(\frac{1}{a}+\frac{2}{b}+\frac{3}{c})=30$. Tìm giá trị lớn nhất của biểu thức:

$P=\frac{b+2c-7\sqrt{72a^2+c^2}}{a}$.

Bài 48: Cho $a,b,c$ là các số thực dương. Tìm giá trị nhỏ nhất của:

$P=\sqrt{\frac{a}{b+c}}+\sqrt{\frac{b}{c+a}}+\sqrt{\frac{c}{b+a}}+\frac{\sqrt{2}(a+b+c)}{\sqrt{ab}+\sqrt{bc}+\sqrt{ca}}$.


Bài viết đã được chỉnh sửa nội dung bởi tritanngo99: 20-09-2016 - 17:07


#98
MrS

MrS

    Binh nhất

  • Thành viên
  • 29 Bài viết

Tiếp theo:

Bài 47: Cho $a,b,c$ là các số thực dương thỏa mãn: $(3a+2b+c)(\frac{1}{a}+\frac{2}{b}+\frac{3}{c})=30$. Tìm giá trị lớn nhất của biểu thức:

$P=\frac{b+2c-7\sqrt{72a^2+c^2}}{a}$.

Bài 48: Cho $a,b,c$ là các số thực dương. Tìm giá trị nhỏ nhất của:

$P=\sqrt{\frac{a}{b+c}}+\sqrt{\frac{b}{c+a}}+\sqrt{\frac{c}{b+a}}+\frac{\sqrt{2}(a+b+c)}{\sqrt{ab}+\sqrt{bc}+\sqrt{ca}}$.

Bài 48:

Không mất tổng quát giả sử c=min(a, b ,c).

Trước tiên ta CM bổ đề quen thuộc: $\sqrt{\frac{a}{b+c}}+\sqrt{\frac{b}{c+a}}\geq 2\sqrt{\frac{a+b}{a+b+2c}}$ ( có thể dùng Holder để chứng minh)

Lại theo AM - GM: $\sqrt{ab}+\sqrt{bc}+\sqrt{ca}\leq \frac{a+b}{2}+\sqrt{2c(a+b)}$

Đặt: $\sqrt{\frac{2c}{b+a}}=t$ với t thuộc (0;1]

Bài toán đưa về tìm Min của: $f(t)=\frac{2}{\sqrt{t^2+1}}+\frac{t}{\sqrt{2}}+\frac{\sqrt{2}(t^2+1)}{2t+1}$

Ta có: $f'(t)=-2t(t^2+1)^{-3/2}+\frac{1}{\sqrt{2}}+2\sqrt{2}(t^2+t-2)(2t+1)^{-2}$ và $f''(t)=2(2t^2-1)(t^2+1)^{-5/2}+18\sqrt{2}(2t+3)^{-3}$

Dễ thấy rằng $f"(t)>0$ với mọi với t thuộc (0;1]

Do đó f'(t) có duy nhất nghiệm t=1.

Khảo sát ta được $Minf(t)=f(1)=\frac{5\sqrt{2}}{2}$

Vậy Min P = $\frac{5\sqrt{2}}{2}$. Đẳng thức xảy ra khi a=b=c


Bài viết đã được chỉnh sửa nội dung bởi MrS: 20-09-2016 - 18:31


#99
MrS

MrS

    Binh nhất

  • Thành viên
  • 29 Bài viết

Bài 48:

Không mất tổng quát giả sử c=min(a, b ,c).

Trước tiên ta CM bổ đề quen thuộc: $\sqrt{\frac{a}{b+c}}+\sqrt{\frac{b}{c+a}}\geq 2\sqrt{\frac{a+b}{a+b+2c}}$ ( có thể dùng Holder để chứng minh)

Lại theo AM - GM: $\sqrt{ab}+\sqrt{bc}+\sqrt{ca}\leq \frac{a+b}{2}+\sqrt{2c(a+b)}$

Đặt: $\sqrt{\frac{2c}{b+a}}=t $ thì t thuộc (0;1]

Bài toán đưa về tìm Min của: $f(t)=\frac{2}{\sqrt{t^2+1}}+\frac{t}{\sqrt{2}}+\frac{\sqrt{2}(t^2+1)}{2t+1}, 0

Ta có: $f'(t)=-2t(t^2+1)^{-3/2}+\frac{1}{\sqrt{2}}+2\sqrt{2}(t^2+t-2)(2t+1)^{-2}$ và $f''(t)=2(2t^2-1)(t^2+1)^{-5/2}+18\sqrt{2}(2t+3)^{-3}$

Dễ thấy rằng $f"(t)>0$ với mọi t thuộc (0;1]

Do đó f'(t) có duy nhất nghiệm t=1.

Khảo sát ta được $Minf(t)=f(1)=\frac{5\sqrt{2}}{2}$

Vậy Min P = $\frac{5\sqrt{2}}{2}$. Đẳng thức xảy ra khi a=b=c



#100
hanguyen445

hanguyen445

    Thượng sĩ

  • Thành viên
  • 240 Bài viết

Tiếp theo:

Bài 47: Cho $a,b,c$ là các số thực dương thỏa mãn: $(3a+2b+c)(\frac{1}{a}+\frac{2}{b}+\frac{3}{c})=30$. Tìm giá trị lớn nhất của biểu thức:

$P=\frac{b+2c-7\sqrt{72a^2+c^2}}{a}$.

Bài 48: Cho $a,b,c$ là các số thực dương. Tìm giá trị nhỏ nhất của:

$P=\sqrt{\frac{a}{b+c}}+\sqrt{\frac{b}{c+a}}+\sqrt{\frac{c}{b+a}}+\frac{\sqrt{2}(a+b+c)}{\sqrt{ab}+\sqrt{bc}+\sqrt{ca}}$.

Bài tập 47:

Khai thác giả thiết tìm ra điều đặc biệt bài toán:

\[\left( {3a + 2b + c} \right)\left( {\frac{1}{a} + \frac{2}{b} + \frac{3}{c}} \right) = 30\]

 \[ \Leftrightarrow 3a\left( {\frac{2}{b} + \frac{3}{c}} \right) + 2b\left( {\frac{1}{a} + \frac{3}{c}} \right) + c\left( {\frac{1}{a} + \frac{2}{b}} \right) = 20\]

\[ \Leftrightarrow \frac{{6a + 2c}}{b} + \frac{{9a + 6b}}{c} + \frac{{2b + c}}{a} = 20 \Leftrightarrow \frac{{2\left( {3a} \right) + 2c}}{b} + \frac{{3\left( {3a} \right) + 3\left( {2b} \right)}}{c} + \frac{{\left( {2b + c} \right)}}{a} = 20\]

Đặt $x=3a;y=2b;z=c$ .Khi đó ta có bài toán:

Cho $x,y,z>0$: \[2.\frac{{2x + 2z}}{y} + \frac{{3\left( {x + y} \right)}}{z} + 3.\frac{{y + z}}{x} = 20\]. 

Tìm Max :

\[P=\frac{3}{2}.\frac{{y + 4z - 14\sqrt {8{x^2} + {z^2}} }}{x}\]

 Dễ nhận thấy giả thiết đối xứng 2 biến $x,z$ và dựa vào cấu trúc P dự đoán dồn biến về $t=\dfrac{z}{x}$. Do đó dự đoán điểm rơi $x=z$

\[2.\frac{{2x + 2z}}{y} + \frac{{3\left( {x + y} \right)}}{z} + 3.\frac{{y + z}}{x} = 20\]

\[ \Leftrightarrow \frac{{4\left( {x + z} \right)}}{y} + 3\left( {\frac{x}{z} + \frac{z}{x}} \right) + 3y\left( {\frac{1}{z} + \frac{1}{x}} \right) = 20 \ge \frac{{4\left( {x + z} \right)}}{y} + 6 + \frac{{12y}}{{x + z}} \Leftrightarrow 14 \ge \frac{{4\left( {x + z} \right)}}{y} + \frac{{12y}}{{x + y}}\]

$\Leftrightarrow 2t+\dfrac{6}{t}\le 7$ với $t=\dfrac{x+z}{y}$ \[ \Leftrightarrow 2{t^2} - 7t + 6 \le 0 \Leftrightarrow \frac{3}{2} \le t \le 2 \Leftrightarrow \frac{3}{2} \le \frac{{x + z}}{y} \le 2 \Leftrightarrow y \le \frac{{2\left( {x + z} \right)}}{3}\]

Suy ra: $P=\dfrac{3y}{2x}+\dfrac{6z}{x}-21\sqrt{8+\dfrac{z^2}{x^2}}\le\dfrac{x+z}{x}+\dfrac{6z}{x}-21\sqrt{8+\dfrac{z^2}{x^2}}$

$=1+\dfrac{7z}{x}-21\sqrt{8+\dfrac{z^2}{x^2}}=1+7t-21\sqrt{8+t^2}=f(t)$ với $t=\dfrac{z}{x}>0$

$f'(t)=7-\dfrac{21t}{8+t^2}=0\Leftrightarrow t=1$

Lập bảng biến thiên suy ra: $P\le f(1)=-55$. Vậy Min P=-55 khi $x=z;y=\dfrac{4}{3}x$


Bài viết đã được chỉnh sửa nội dung bởi hanguyen445: 20-09-2016 - 22:59

Đề thi chọn đội tuyển  HSG:

http://diendantoanho...date-2016-2017/

Topic thảo luận bài toán thầy Hùng:

http://diendantoanho...topicfilter=all

Blog Thầy Trần Quang Hùng

http://analgeomatica.blogspot.com/

Hình học: Nguyễn Văn Linh

https://nguyenvanlin...ss.com/2016/09/

Toán học tuổi trẻ:

http://www.luyenthit...chi-thtt-online

Mathlink:http://artofproblemsolving.com

BẤT ĐẲNG THỨC:

http://diendantoanho...-đẳng-thức-vmf/

http://diendantoanho...i-toán-quốc-tế/

 





1 người đang xem chủ đề

0 thành viên, 1 khách, 0 thành viên ẩn danh